Sei sulla pagina 1di 518

Physics Graduate School Qualifying Examination

Spring 2005

Part I
________________________________________________________________________

Instructions: Work all problems. This is a closed book examination. Calculators may
not be used. Start each problem on a new pack of correspondingly numbered paper
and use only one side of each sheet. Place your 3-digit identification number in the
upper right hand corner of each and every answer sheet. All sheets, which you
receive, should be handed in, even if blank. Your 3-digit ID number is located on your
envelope. All problems carry the same weight.
________________________________________________________________________

1. A uniform flexible rope is suspended above a scale, with the bottom of the rope
just touching the scale (gravity points downward). The rope has a length L and a
total mass of M. The mass is uniformly distributed along its length.

gravity
L

Scale

The rope is released. After a length x<L has fallen onto the scale, what does the
scale read? Assume the scale can measure the force applied to it instantaneously.
Also assume the rope is relatively short so that the force of gravity does not vary over
its length. (Hint: the force exerted by the rope on the scale has two components).

1
2. One end of a smooth thin rigid rod is attached at 90 degrees to a vertical axle. A
motor drives the axle so that the rod rotates in a horizontal plane at a constant
angular velocity of ω. A small bead of mass m, with a hole through its center,
slides without friction on this rod.
y

ω ω
m
m
Side view
θ
x
Top view

a) Find the equations of motion for the bead in the polar coordinates r, θ.

b) Is the vertical component (z-component) of the angular momentum


conserved? Explain your answer.

c) Solve for the radial motion of the bead r(t) if at time t=0 the bead is rotating
with the rod with radial velocity r&(t = 0) = 0 and is located at r(t=0)=ro.

2
3. A large metal sphere of radius RL is connected to a small metal sphere of radius RS
by a thin wire. A total charge of QT is placed on the spheres (assume that all the
charge is on the spheres and none is on the wire; the wire maintains the spheres at
the same potential).

RL
RS

a) What fraction of the total charge is on each sphere?

b) Compare the electric fields on the surface of each sphere. What is the
ratio of these electric fields, i.e. what is ES/EL?

(Assume to first order that the capacitance of each sphere is independent


of the presence of the other sphere. That is, the separation of the two
spheres is much larger than RL or RS)

3
4. Consider a permanently polarized dielectric cube with the origin of the
coordinates at the center of the cube. The cube has a side of length a. The
r r r
permanent polarization of the dielectric is P = cr . The vector r is the radius
r
vector from the origin of the coordinates to the point r ( x, y, z ) . There are no free
charges in the system. Compute the bound charges of the system.

4
5. The electron neutrino is thought to be nearly massless (it may have no mass; the
issue is not definitively settled). As it turns out, one way to place an upper limit
on the mass is from the detection of the neutrinos from a supernova. Assume that
your detector measures a series of neutrinos within a time interval Δt with mean
energy E and energy spread (variance) of ΔE.

a) If the neutrino is massless, what can be said qualitatively about the


arrival times of the neutrinos with respect to their energies, assuming
they were emitted at the same time?

b) If the neutrinos have mass, what can be said qualitatively about the
arrival times of the neutrinos with respect to their energies, assuming
they were emitted at the same time?

c) Derive an expression for the mass of the neutrino, given that the
supernova lies at a distance d away from the earth. Express your
answer as a function of ΔE and Δt. (Assume that the neutrino rest
mass is much less than its total energy, i.e. E >>mc2).

5
6. A source illuminates equally two slits (1) and (2). The source can emit either
“classical” or quantum particles. Two detectors, (A) and (B), are located directly
behind slits (A) and (B) in the viewing screen, respectively (the two detectors are
separated by a wall so no particle detected at one detector can also be detected at
the other). Finally, a barrier screen is placed so that particles entering slit (1)
cannot enter detector (A).
barrier screen

(A)
(A) detector
(1)

source (2)
(B) detector
(B)

The wavefunctions at the detector locations for particles going through slit (1) and
slit (2) are φ1 and φ2. These wavefunctions are orthogonal and when normalized
individually they have the values shown in the table below.
Location
(A) (B)
φ1 0 1
10
e − iπ / 2
φ2 1
10
e − iπ / 2 1
10
e + iπ / 2

a) Write the correctly normalized wavefunction for the two-slit system.


b) If 20 “classical” particles (non-interfering) are passed through the slits
(equally likely through (1) or (2)), what is the expected number of events
observed in each detector (using the above table)?
c) If 20 quantum particles are passed through the slits (again, equally likely
through (1) or (2)), what is the expected number of events observed in
each detector (using the above table)?
d) For quantum particles, is interference observed in either detector? From
the detection of a quantum particle in detector (A) can you tell whether
the particle went through slit (1) or slit (2)? If it were detected in (B)?

6
7. One mole of a monatomic perfect gas initially at temperature T0 expands from
volume V0 to 3V0 at constant temperature. Calculate the work of expansion and
the heat absorbed by the gas.

7
8. Consider a system with a large number N of distinguishable, noninteracting
particles at rest. Each particle has 2 nondegenerate energy levels with energies of
0 and E >0, respectively. Let E/N be the mean energy per particle as N ⇒ ∞ .

Energy Levels
E

(a) What is the maximum possible value of E/N if the system is not necessarily in
thermodynamic equilibrium?

(b) What is the maximum possible value of E/N if the system is in


thermodynamic equilibrium?

(c) Compute the entropy per atom S/N as a function of E/N.

8
Physics Graduate School Qualifying Examination
Spring 2005

Part II
________________________________________________________________________

Instructions: Work all problems. This is a closed book examination. Calculators may
not be used. Start each problem on a new pack of correspondingly numbered paper
and use only one side of each sheet. Place your 3-digit identification number in the
upper right hand corner of each and every answer sheet. All sheets, which you
receive, should be handed in, even if blank. Your 3-digit ID number is located on your
envelope. All problems carry the same weight.

________________________________________________________________________

9. A small mass m is located at a radius b from the center of a turn-table that is


initially at rest. The turn-table is horizontally-oriented on the surface of the earth
(i.e. the force of gravity points directly down, − ẑ , perpendicular to the surface of
the turn-table). At the time t=0, the turn-table starts to accelerate with constant
angular acceleration α. The coefficient of static friction between the mass and the
surface of the turn-table is μ .

z
α
b
m

a) At the moment the mass starts to slip, in the rotating frame, write down
all of the forces on the mass.

b) At what time will the mass start to slip?


r v r r r r r r
(Hint: a = ω& × r '+2ω × v '+ω × (ω × r ' )
Where the primed system is the accelerated or rotating system. The dot as usual
denotes a time derivative.)

9
10. A binary star system (consisting of two stars with masses m1 and m2) has a total
mass M=m1+m2. The two stars execute circular orbits about the system’s center
of mass with a period of T= 3years. The radii of the two orbits are l1 and l2,
respectively. The total distance between the two stars is 2 astronomical units.
(one astronomical unit is the distance from the earth to the Sun).

m2

l2

l1
m1

a) What are the orbital velocities of each star?

b) What are the centripetal forces on each star?

c) Express the mass M of the binary star system in terms of the mass of our
Sun, Ms.

10
11. An infinite plane with uniform surface charge density of σ, is lying in the z=0
plane. This infinite plane is moving parallel to itself in the x-direction with
r
velocity v .

σ r
v
y x

r
a. Find the surface current density K (units of C/m/s).
r
b. Find the B field above and below the plane.

Now add a second plane with the same charge density σ. It is parallel to the first
but at a distance s below the first. It moves with same magnitude of velocity |v|
but in the negative x-direction.
r
c. Find the B field between the two planes.
r
d. Find the B field outside the two planes.

e. What is the magnetic force per unit area on the upper plate?

11
12. A metal bar of mass M and length L slides down between two vertical metal posts.
It slides with negligible friction but with good electrical contact. The bar falls in
the gravitational field of the Earth at a constant speed of v. The falling bar and the
vertical posts have negligible electrical resistance, but the base connecting the two
posts is an ohmic resistor with resistance R. Throughout the entire region there is
a uniform magnetic field of magnitude B coming straight out of the page.

v
Bar of mass M

B ( out of page)

Resistance R

a. Calculate the amount of current I running through the resistor.


Indicate the direction of the conventional current.

b. Calculate the constant speed v of the falling bar.

c. Show that the rate of change of the gravitational energy of this system
is equal to the rate of energy dissipated in the resistor.

12
13. Consider a simple 1D harmonic oscillator. Its Hamiltonian is

p 2 mω 2 2
H0 = + x
2m 2

a. The eigenfunction of the ground state wave function can be written as


ψ 0 ( x) = Ne −α
2 2
x /2

Determine N and α.

b. What is the eigenvalue of the ground state?

r
c. At t=0, an electric field E is switched adding a perturbation whose
Hamiltonian has the form
r
H' = e E x

What is the new ground state energy?

d. Assuming that the field is switched on in a time much shorter than


1/ω, what is the probability that the particle stays in the ground state?

π
∫ dxe
−a 2 x 2
(A useful integral: = )
−∞
a

13
14. Compute the energy levels of the hydrogen atom using the Bohr model. Use this
formula to calculate the Kα spectra for atoms with many electrons. That is, use
this formula to calculate the energy of the photon released when an electron from
the n=1 state is removed and an electron from the n=2 state makes the transition
down to the n=1 state. Calculate your answer for Z=2 and for Z=90. Express
your answer in terms of ε 0 the electric permitivity, e the charge of the electron,
and h Planck’s constant.

14
7. A photon of energy Eγ is incident along the positive x-axis on a stationary electron
of mass m at the origin in the lab frame. The goal of this problem is to find the
velocity of the center of mass in the lab frame, vcm. This will be accomplished in
the three following steps:

a. Give the Lorentz transformation for the linear momentum px of the


photon and the electron from the lab frame to the center of mass (cm)
frame symbolically in terms of the velocity of the center of mass vcm in
the lab frame.

b. Find p x (γ ) and p x (e) , the momenta for the photon and the electron,
respectively, in the cm frame in terms of vcm and Eγ .

c. Solve for vcm.

15
8. Two identical finite bodies of constant heat capacity Cv are at temperatures T1 and
T2, respectively. These two bodies are used to run a heat engine. If these bodies
remain at constant volume and they undergo no phase change:

a) Show that the amount of work obtainable is

W = C v (T1 + T2 − 2T f )

where Tf is the final temperature attained by both bodies.

b) Show that when W is a maximum,

T f = T1T2

16
Physics Graduate School Qualifying Examination
Spring 2005

Part II
________________________________________________________________________

Instructions: Work all problems. This is a closed book examination. Calculators may
not be used. Start each problem on a new pack of correspondingly numbered paper
and use only one side of each sheet. Place your 3-digit identification number in the
upper right hand corner of each and every answer sheet. All sheets, which you
receive, should be handed in, even if blank. Your 3-digit ID number is located on your
envelope. All problems carry the same weight.

________________________________________________________________________

1. A small mass m is located at a radius b from the center of a turn-table that is


initially at rest. The turn-table is horizontally-oriented on the surface of the earth
(i.e. the force of gravity points directly down, − ẑ , perpendicular to the surface of
the turn-table). At the time t=0, the turn-table starts to accelerate with constant
angular acceleration α. The coefficient of static friction between the mass and the
surface of the turn-table is μ .

z
α
b
m

a) At the moment the mass starts to slip, in the rotating frame, write down
all of the forces on the mass.

b) At what time will the mass start to slip?


r v r r r r r r
(Hint: a = ω& × r '+2ω × v '+ω × (ω × r ' )
Where the primed system is the accelerated or rotating system. The dot as usual
denotes a time derivative.)
2. A binary star system (consisting of two stars with masses m1 and m2) has a total
mass M=m1+m2. The two stars execute circular orbits about the system’s center
of mass with a period of T= 3years. The radii of the two orbits are l1 and l2,
respectively. The total distance between the two stars is 2 astronomical units.
(one astronomical unit is the distance from the earth to the Sun).

m2

l2

l1
m1

a) What are the orbital velocities of each star?

b) What are the centripetal forces on each star?

c) Express the mass M of the binary star system in terms of the mass of our
Sun, Ms.
3. An infinite plane with uniform surface charge density of σ, is lying in the z=0
plane. This infinite plane is moving parallel to itself in the x-direction with
r
velocity v .

σ r
v
y x

r
a) Find the surface current density K (units of C/m/s).
r
b) Find the B field above and below the plane.

Now add a second plane with the same charge density σ. It is parallel to the first
but at a distance s below the first. It moves with same magnitude of velocity |v|
but in the negative x-direction.
r
c) Find the B field between the two planes.
r
d) Find the B field outside the two planes.

e) What is the magnetic force per unit area on the upper plate?
4. A metal bar of mass M and length L slides down between two vertical metal posts.
It slides with negligible friction but with good electrical contact. The bar falls in
the gravitational field of the Earth at a constant speed of v. The falling bar and the
vertical posts have negligible electrical resistance, but the base connecting the two
posts is an ohmic resistor with resistance R. Throughout the entire region there is
a uniform magnetic field of magnitude B coming straight out of the page.

v
Bar of mass M

B ( out of page)

Resistance R

a) Calculate the amount of current I running through the resistor.


Indicate the direction of the conventional current.

b) Calculate the constant speed v of the falling bar.

c) Show that the rate of change of the gravitational energy of this system
is equal to the rate of energy dissipated in the resistor.
5. Consider a simple 1D harmonic oscillator. Its Hamiltonian is

p 2 mω 2 2
H0 = + x
2m 2

a) The eigenfunction of the ground state wave function can be written as


ψ 0 ( x) = Ne −α
2 2
x /2

Determine N and α.

b) What is the eigenvalue of the ground state?

r
c) At t=0, an electric field E is switched adding a perturbation whose
Hamiltonian has the form
r
H' = e E x

What is the new ground state energy?

d) Assuming that the field is switched on in a time much shorter than


1/ω, what is the probability that the particle stays in the ground state?

π
∫ dxe
−a 2 x 2
(A useful integral: = )
−∞
a
6. Compute the energy levels of the hydrogen atom using the Bohr model. Use this
formula to calculate the Kα spectra for atoms with many electrons. That is, use
this formula to calculate the energy of the photon released when an electron from
the n=1 state is removed and an electron from the n=2 state makes the transition
down to the n=1 state. Calculate your answer for Z=2 and for Z=90. Express
your answer in terms of ε 0 the electric permitivity, e the charge of the electron,
and h Planck’s constant.
7. A photon of energy Eγ is incident along the positive x-axis on a stationary electron
of mass m at the origin in the lab frame. The goal of this problem is to find the
velocity of the center of mass in the lab frame, vcm. This will be accomplished in
the three following steps:

a) Give the Lorentz transformation for the linear momentum px of the


photon and the electron from the lab frame to the center of mass (cm)
frame symbolically in terms of the velocity of the center of mass vcm in
the lab frame.

b) Find p x (γ ) and p x (e) , the momenta for the photon and the electron,
respectively, in the cm frame in terms of vcm and Eγ .

c) Solve for vcm.


8. Two identical finite bodies of constant heat capacity Cv are at temperatures T1 and
T2, respectively. These two bodies are used to run a heat engine. If these bodies
remain at constant volume and they undergo no phase change:

a) Show that the amount of work obtainable is

W = C v (T1 + T2 − 2T f )

where Tf is the final temperature attained by both bodies.

b) Show that when W is a maximum,

T f = T1T2
Physics Graduate School Qualifying Examination
Fall 2005

Part I
________________________________________________________________________

Instructions: Work all problems. This is a closed book examination. Calculators may
not be used. Start each problem on a new pack of correspondingly numbered paper
and use only one side of each sheet. Place your 3-digit identification number in the
upper right hand corner of each and every answer sheet. All sheets, which you
receive, should be handed in, even if blank. Your 3-digit ID number is located on your
envelope. All problems carry the same weight.
________________________________________________________________________

1. A marble of mass m falls under the influence of gravity into a beaker so that it
enters the fluid in the beaker with an initial velocity of vo. The fluid in the beaker
is viscous and so produces a drag force linear in velocity (characterized by the
parameter α) on the marble of
r r
F = − mα v
Assume that the buoyant force can be neglected and:

(a) Write the equation of motion for the marble;


(b) Solve the equation of motion for the location of the marble as a function of
time.

gravity r
v

1
2. A long thin tube of negligible mass is pivoted so it may rotate freely without
friction in a horizontal plane (see side view). A thin rod of mass M and length L
lies in the tube and can slide without friction in the tube. Recall that the moment
of inertia of a thin rod about an axis perpendicular to rod’s long axis and through
its center is ML2/12.

rotation
axis
M r M θ
r
L
Side view
Top view

a) Find the equations of motion for the thin rod in terms of the angle of the tube
θ and the radial position of the center of the rod r.

b) Find an expresion for the vertical component (z-component) of the angular


momentum. Is it conserved? Justify your answer.

c) Find an expression for the radial velocity r& as a function of r .

2
3. A thin coil with radius a, located at z = 0 in the x-y plane, consists of n turns of
wire and carries a current I(t) that is initially zero and then increases linearly to
the value I0 over a time τ :

⎧0 t<0

I (t ) = ⎨ I 0 t / τ 0 < t <τ
⎪I t >τ
⎩ 0

.
The dipole field created by this loop near the origin has a z-component that can be
approximated by

⎛ a 2 nμ I (t ) ⎞
B z = −⎜ 0 ⎟
⎜ 3 ⎟
⎝ 4r ⎠

Another small, single loop of wire of radius b<<a , is also located in the x-y plane and is
concentric with the larger coil. This wire has resistivity ρ and total mass m.

(a) Calculate the magnitude of the induced electric field at the location of the small
loop.

(b) Calculate the magnitude of the electric current that is made to flow in the loop.

z
n loops radius a
single loop radius b

3
4. Two solid metal balls 1 and 2 of radii R1 and R2 (R1 > R2), respectively, are far
from each other so that they do not polarize each other. Excess charge Q is
supplied to the larger sphere

Q
R1
R2

2
1
r
(a) What is the electric field E1c in the center of the metal ball 1?
r
(b) What is the electric field vector E1o just outside the metal ball 1?

(c) What is the potential V1c in the middle of the metal ball 1?

(d) The metal balls are placed in vacuum and then connected to each other using a
long very thin metal wire so that charge may flow from one to the other. Assume no
significant charge remains on the wire. After the system has reached steady state,
the wire is removed. The atmosphere is then let into the chamber and it becomes
possible for a spark to occur from the surface of one of the balls. This would
happen if the electric field exceeded the breakdown field for the atmosphere, Ecrit.
Calculate the minimum amount of excess charge Q that had to be placed initially on
the first ball for the spark to occur in terms of Ecrit.

4
5. Provide brief answers to five of the following:

(a) What was the main result or significance of the Millikan oil-drop
experiment?
(b) What was the main result or significance of the Stern-Gerlach
experiment?
(c) What was the most significant discovery made by deep inelastic
electron-proton (e-p) scattering experiments?
(d) What are two experimental tests of General Relativity?
(e) How is the Hubble constant defined?
(f) What was the main result or significance of the Franck-Hertz
experiment?
(g) What did Rutherford’s scattering experiments tell us about atoms?
(h) Describe two experiments. One in which light behaves like a wave
and the other where it behaves like a particle.

5
6. The quantum mechanical wavefunction for a particle (or bead) of mass m
confined to a hoop of radius r is r

ψ n = Aeinφ n = 0,±1,±2,... m

The Schrodinger wave equation for this system is


φ
h2 1 d 2
− ψ = Eψ
2 m r 2 dφ 2

a) Write the correctly normalized wavefunctions for this system.


b) Find the energy of state n.
c) If the hoop is placed vertically in a gravitational field, what is the first
order shift in the particle’s energy due to this perturbation? Also for
what eigenstates is this perturbation approximation valid?

gravity
r
φ

6
7. A lambda hyperon Λ0 (mass = 1115.7 MeV/c2) at rest in the lab frame decays into
a neutron n (mass = 939.6 MeV/c2) and a pion π0 (mass = 135.0 MeV/c2):

Λ0 → n + π0

What are the kinetic energies (in the lab frame) of the neutron and pion after the
decay?

7
8. A system consists of N pairs of non-interacting, spin-1/2 particles with magnetic
moment µ. It is initially immersed in a uniform magnetic field Hi while it is in thermal
equilibrium with a reservoir at temperature Ti. While remaining in thermal equilibrium
with the reservoir, the magnetic field is increased to a value of Hf . At this point the
system is isolated from the reservoir and the magnetic field returns adiabatically to Hi.
The system is then brought into thermal equilibrium with a second reservoir at
temperature Tf .

(a) How much work is done on the system by the magnetic field?
(b) How much heat is transferred between the two reservoirs?

8
Physics Graduate School Qualifying Examination
Fall 2005

Part II
________________________________________________________________________

Instructions: Work all problems. This is a closed book examination. Calculators may
not be used. Start each problem on a new pack of correspondingly numbered paper
and use only one side of each sheet. Place your 3-digit identification number in the
upper right hand corner of each and every answer sheet. All sheets, which you
receive, should be handed in, even if blank. Your 3-digit ID number is located on your
envelope. All problems carry the same weight.

________________________________________________________________________

1. Consider a sphere of gas of radius R, mass M, and uniform density ρ. A point


mass m << M is orbiting the gas cloud in a stable circular orbit of radius r.
Assume gravitational forces only and no friction or drag.

a) What is the velocity of the point mass for r > R and r < R?

b) Derive a relationship between the orbital radius r and the orbital period
P of the point mass for r > R and r < R.
2. A disk (yo-yo) of radius R and mass m is attached to a weightless cord as shown.
At time zero, the disk is released and a force upward is applied to the upper end of
the cord so that the center of mass of the disk does not move up or down. What
must be the minimum length, L, of the cord wound around the disk at time zero if
you wish to keep it hovering this way until time t after you release it?

L=?

R R
m m
floor
t=0 t=t
3. The speed of an electromagnetic wave propagating
in a coaxial cable with inner radius r and outer
radius R is:

1
v=
LC r
R

where L is the cable’s inductance per unit length and C is the cable’s capacitance per unit
length. For a cable filled with a dielectric material with dielectric constant εm and
magnetic permeability μm find the speed of the E&M waves in the cable by calculating L
and C.
4. Consider an ideal cylindrical solenoid of length L and radius a=L/2 on which a
thin wire has been wrapped a total of N turns. A steady current of I flows through
the wire. Assume that the wires are wound so tightly that the solenoid can be
thought of as a collection of N parallel current loops. Use the Biot-Savart law:

r r μ r r
I ' ds '× R
B(r ) = o
4π ∫ R3
C'

to find the induced magnetic field at the exact center of the solenoid.
5. Find the rms position uncertainty Δx of a quantum particle in the state with the
wavefunction

2 2
e − x / 2a
1
ψ o ( x) =
a π
see the integral tables for a useful integral.
6. Consider two identical particles of mass m and spin 1/2 . They interact via a
potiential given by
g
V= σˆ1 ⋅ σˆ 2
r

where g>0 and σ̂ j are Pauli spin matrices which operate on the spin of particle j.

(a) Construct the spin eigenfunctions for the two particle states. What is the
expectation value of V for each of these states?

(b) Give eigenvalues of all the bound states.


7. Consider a blob of luminous matter moving along a direction x̂ at an angle θ to
your line of sight at a relativistic speed v.

Observer
x
v
Blob θ
d

a) According to you, the stationary observer at a distance d, what is the


apparent transverse velocity of the blob? Hint: you must take into
account light travel time effects.

b) If the blob emits light monochromatically at frequency ωo in its rest


frame, what frequency will you measure in the lab frame?
8. A quantity of one mole of an ideal gas in a volume V is allowed to freely expand
into an equal volume so that the volume of the gas doubles (see below). The
process is adiabatic. What is the change in entropy of the gas?

Before free expansion

V V

After free expansion

V V
Physics Graduate School Qualifying Examination
Spring 2006

Part I
________________________________________________________________________

Instructions: Work all problems. This is a closed book examination. Calculators may
not be used. Start each problem on a new pack of correspondingly numbered paper
and use only one side of each sheet. Place your 3-digit identification number in the
upper right hand corner of each and every answer sheet. All sheets, which you
receive, should be handed in, even if blank. Your 3-digit ID number is located on your
envelope. All problems carry the same weight.
________________________________________________________________________

1. A smooth hemisphere is fixed to a horizontal plane on the surface of the earth.


Under the influence of gravity, a block of mass m slides without friction down the
sphere. The block starts at rest from the top of the sphere. R is the radius of the
sphere and θ is the polar angle.

R
θ v

Find the value of θ at which the block leaves the sphere and its speed v at that
instant.

1
2. A block of mass m oscillates horizontally, sliding on a frictionless surface. The
ideal spring has a spring constant of k. The maximum displacement of the block
from the equilibrium position is A.

Find the maximum velocity of the block in terms of k, m, and A.

2
3. Consider a spherical shell of dielectric material with a permanent polarization,

r r k rr
P(r ) =
rr
.
where k is a constant.
r The inner radius of the shell is a and the outer radius is b. The
position vector, r , has its origin at the center of the spherical shell and the magnitude of
the position vector is r. r
r P r
P P
r r
P P
a
r r
P b P
r r
P P
r r
P r P
P

(a) Calculate the bound charge distribution for this shell.

(b) Calculate the electric field produced by the shell for the regions:

I: r < a
II: a < r < b
III: r > b

3
4. Consider the infinite straight wire and the rectangular wire loop shown below.
The loop and the wire lie in the same plane. The top edge of the loop lies a
distance a from the wire while the bottom edge is a distance b from the wire. The
long side of the loop is a length L.

I
a

(a) Given a constant current I flowing through the straight wire, what is the flux
through the loop?

(b) What is the mutual inductance of this system?

4
5. Two isolated gamma rays are emitted in opposite directions in the decay of
positronium. The gamma rays are oppositely polarized. The two different
polarization states are denoted L for left circular polarization and R for right
circular polarization. Please use bra and ket notation in your answers to the
following.

A B
γA γB

(a) Write the normalized, symmetric quantum mechanical polarization


wave function for the pair of correlated gamma rays.
(b) The detector A measures linear polarization in the X and Y basis
such that
R = 1
2
(X +iY ) L = 1
2
(X −iY )
if γA is measured to be in the state Y , what is the probability that the
detector B will measure γB in an L or R state, given that detector
B measures circularly polarized states.

5
6. The particle of mass m is confined in a one-dimensional, impenetrable box with
walls at x=0 and x=+L . The particle is in its ground state.

x=0 x=+L x

a) Using Schrodinger’s equation and the trial wave function,

ψ ( x ) = A cos( kx ) + B sin( kx )

show that the eigenfunctions of the particle are

ψ n (x) = L
( )
2 sin n π x
L

where n= 1, 2, ...

b) Compute the energies of the eigenfunctions.

c) Show that n=1 is the ground state.

d) The right wall of the box is moved instantaneously to x=2L. Calculate the
probability that the particle jumped from the initial ground state of the old
box to the first excited state of the new, expanded box.

x=0 x=+L x=+2L x

Some possibly useful integrals:


x 1
cos 2 ( ax ) =+ cos( ax ) sin( ax )
2 2a


x 1
sin 2 (ax ) = − cos( ax ) sin( ax)
2 2a

6
7. A space station consists of a large cylinder of radius Ro filled with air. The
cylinder spins about its symmetry axis at an angular velocity of Ω providing a
centripetal acceleration of g at the rim of the cylinder (at Ro).

Ro g

If temperature T is constant inside the station, what is the ratio of the air
pressure Pc at the center of the cylinder to the air pressure at the rim Pr? (Treat
the air as an ideal gas with each gas particle having a mass of m.)

7
8. A classical system of N distinguishable, noninteracting particles of mass m is placed in
a 3D harmonic potential,

x2 + y2 + z 2
U (r ) = η
2V 2 / 3
where V is a volume and η is a constant with units of energy.

(a) Find the partition function and the Helmholtz free energy of the system.

(b) Find the entropy, the internal energy, and the total heat capacity at constant
volume for the system.

A possibly useful integral:

∞ 2 1 ⋅ 3 ⋅ 5 ⋅ ... ⋅ (2n − 1) π
2n − ax
∫x e dx = ; for integer n.
0 2 n +1 a n a

8
Physics Graduate School Qualifying Examination
Spring 2006

Part II
________________________________________________________________________

Instructions: Work all problems. This is a closed book examination. Calculators may
not be used. Start each problem on a new pack of correspondingly numbered paper
and use only one side of each sheet. Place your 3-digit identification number in the
upper right hand corner of each and every answer sheet. All sheets, which you
receive, should be handed in, even if blank. Your 3-digit ID number is located on your
envelope. All problems carry the same weight.

________________________________________________________________________

1. A bead of mass m slides without friction on a straight thin wire. The lower
end of the thin wire is attached to the thin, vertical axle of a motor that is
rotating at constant angular velocity ω. The angle between the thin wire and
the axle, α, is fixed. r is the variable distance along the wire to the bead from
the point where the wire’s lower end is attached to the axle. The bead is under
the influence of earth’s gravity.
z
ω
m
α r

y
θ
x

a) Find the kinetic energy of the bead in the stationary frame (coordinates x,
y, z) in terms of r, dr/dt, ω, and α.

b) Find the equation of motion of the bead in terms of r and its derivatives
including the effects of gravity.
2. A block of mass m hangs from one end of a massless cord. The other end of cord
is wrapped around a wheel of radius R and of moment of inertia I. The wheel is
free to rotate about a fixed, frictionless horizontal axis through its center. The
system starts from rest and the cord unwinds with no slippage under the influence
of earth’s gravity.

Find the acceleration, a, of the block.


3. Consider an infinitely long straight wire that carries a current I.

a) Derive the magnetic field strength as a function of r, the radial


distance from the wire.

b) A thin rod of length L oriented parallel to the wire moves toward it at


a speed v << c.
I

What is voltage difference between the ends of the rod, as a function


of r?
r
4. Consider a particle with charge q and acceleration u& in the plane of the page as
shown below. The particle is moving non-relativistically such that the radiation
r r
field is given by (note Erad , n̂ , and u& are in the plane of the page)

r
r q ⎡ nˆ × (nˆ × u& ) ⎤
Erad = ⎢ ⎥
4πε o c 2 ⎣ R ⎦

r nˆ r
Brad = × Erad
c

where R and θ are the distance and direction to the point P where the fields are
being measured.
r
u&

P
r
E rad

R
θ

particle

a) Calculate the magnitude of the Poynting vector. In what direction is


it?

b) By integrating over all solid angles, calculate the total power emitted
by the particle.

∫ sin
3
Useful Integral: ( x)dx = − 13 cos( x)[sin 2 ( x) + 2]
5. A quantum mechanical particle with mass m and energy E approaches the
potential drop shown below.
m
0 ⎧⎪ 0 ; x ≤ 0
V ( x) = ⎨
-Vo ⎪⎩− Vo ; x > 0
x=0

What is the probability it will be reflected by the potential?


6. A source illuminates two slits in a screen equally. On the other side of the screen,
two detectors A and B are located at x=A and x=B, respectively (see below).
x
(1) A

source
(2)
B
screen

The wavefunction for a particle going through slit (1) with slit (2) closed is φ1.
Similarly φ2 is the wavefunction for a particle going through slit (2) with slit (1)
closed. These wave functions are orthogonal and when normalized individually they
have the values shown below at the locations of the detectors:

Wave Function Location A Location B


φ1 1 1 e −i π / 2
5 5
φ2 0 1 e + iπ / 2
5

a) Write the correctly normalized wave function for the two-slit system.

b) If 10 quantum mechanical particles are passed through the slits (each with
equal probability of passing through slit (1) or (2)). What is the expected
number of events observed in each detector, using the wave function values in
the table?
7. Consider an atom in a crystal that is described as a particle of mass m in an
infinite square well potential,

⎧⎪ 0 ; 0 < x < L
V ( x) = ⎨
⎪⎩∞ ; otherwise

(a) What are the allowed energies, En, of such a particle?

(b) If the atom is in thermal equilibrium at a temperature T, what are the


probabilities of finding the atom in the ground state and in the first
excited state? Assume that the probability of finding the atom in any
higher state is negligibly small.

(c) Suppose a crystal of N such atoms was in thermal equilibrium at


temperature T. What is the average energy of the system? Still
assume that the probability of finding the atom in any higher state is
negligibly small.
8. The Fermilab Tevatron is a synchrotron that stores beams containing on the order
of N=1012 protons and each proton has an energy of approximately E=1TeV (1012
eV).

(a) To safely remove the beam from the synchrotron, the beam is steered
into a cylindrical block of graphite, d=15 cm in diameter and l=4.5 m
in length. Calculate the increase in temperature of the graphite block
when the beam is dumped and completely absorbed in the graphite
block. The density of graphite is ρ=2.265 g/cm3 and its specific heat is
C=720 J/K/kg. Solve algebraically before making numerical estimate.
Note one eV is equal to 1.6x10-19J.

(b) The Large Hadron Collider (LHC) circulates proton beams that
contain on the order of 1014 protons each with an energy of
approximately 7 TeV. The cylindrical graphite beam dump is larger
for this synchrotron, 1.5m in diameter and 10m in length. Calculate
the increase in temperature of this graphite beam dump when the
LHC’s beam is dumped into it.
Physics Graduate School Qualifying Examination
Fall 2006

Part I
________________________________________________________________________

Instructions: Work all problems. This is a closed book examination. Calculators may
not be used. Start each problem on a new pack of correspondingly numbered paper
and use only one side of each sheet. Place your 3-digit identification number in the
upper right hand corner of each and every answer sheet. All sheets, which you
receive, should be handed in, even if blank. Your 3-digit ID number is located on your
envelope. All problems carry the same weight.
________________________________________________________________________

1. A bead of mass m slides without friction on a rotating wire hoop of radius a. The
axis of rotation is through a vertical diameter of the hoop as shown in the figure.
The earth’s gravity acts in the vertical direction. The constant angular velocity of
the hoop is ω.

a
θ
m

(a) Write the Langrangian of the bead.


(b) Find the equation of motion.
(c) Is the vertical component of the angular momentum conserved? Why?

1
2. Kepler’s second law states: The radius vector from the Sun to a planet sweeps out
equal areas in equal intervals of time. Using Newtonian physics, prove that
Kepler’s second law is correct for an elliptical orbit.

r r r
r + Δr Δr
ΔA
r
r

2
3. A toroidal coil has a rectangular cross section with inner radius a and outer radius
b, and height h. The toroid coil consists N rectangular turns or loops whose
planes are vertical. One such rectangular loop is highlighted in the cross-section
view below.

a
h
b
.

Toroidal Coil
Cross Section of
Toroidal Coil
Find the self inductance of the toroidal coil.

3
4. A ring of radius R carries constant conventional current I in the direction shown. Its
axis lies along the x axis. The ring is moving to the left along the x axis with uniform
speed v<<c. At time t=0, the center of the ring is at the coordinate (L, 0, 0).

y
I

R
y0
x
v

I
L

(a) Starting from Biot-Savart’s law, calculate the magnetic field at time t=0 at the
origin of the coordinate system (0, 0, 0).

(b) Calculate the electric field at time t=0 (magnitude and direction) at the location
(0, y0, 0) that is close to the axis y0<<L . If you make any simplifying assumptions or
approximations, state them explicitly.

4
5. Cold neutrons can bounce off a surface with no loss of energy. Consider a cold
neutron bouncing up and down off a horizontal surface in earth’s gravitational field
near the surface of the earth. Thus the cold neutron is trapped in a potential
V ( z ) = mgz where z is the height above the surface.
z

Use the Bohr-Sommerfeld quantization rules or dimensional analysis (or other


methods) to find the allowed quantization energy levels for this bouncing cold
neutron. [If necessary you can assume En~n2/3].

5
6. Consider the double delta function potential.

V ( x ) = − α [δ ( x + a ) + δ ( x − a ) ]

where α and a are positive constants..

a) Sketch this potential.

b) Given that a particle in this potential has only two bound states, sketch the
associated wavefunctions.

h2
c) Find the allowed bound state energies for a particle of mass m for α = .
ma

⎛ dψ (x + ε ) dψ (x − ε ) ⎞ +ε
∫− ε V ( x )ψ ( x ) dx ]
2m
[Hint: for small ε, ⎜ − ⎟ = 2
⎝ dx dx ⎠ h

6
7. Find the efficiency of an ideal engine that works according to the following cycle.

P
Isotherm at
temperature T1
P2
Isotherm at
P1 temperature T2

V2 V1 V

Express your answer in terms of the volumes V1 and V2 and the heat capacities at
constant pressure and volume Cp and Cv. Recall that the efficiency may be defined
as

Net work done over full cycle


η=
Heat absorbed along isotherm T1

7
8 . A very long cylindrical solenoid has a radius R. It is moving at relativistic speed
v perpendicular to its axis in the lab frame. The solenoid produces a uniform
magnetic field B in its rest frame.
R

(a) In the solenoid rest frame, compute the magnetic flux intercepted by a
horizontal cross section of the solenoid.

(b) In the lab frame, compute the magnetic flux intercepted by a horizontal cross
section of the solenoid. Compare your answer to part (a).

8
Physics Graduate School Qualifying Examination
Fall 2006

Part II
________________________________________________________________________

Instructions: Work all problems. This is a closed book examination. Calculators may
not be used. Start each problem on a new pack of correspondingly numbered paper
and use only one side of each sheet. Place your 3-digit identification number in the
upper right hand corner of each and every answer sheet. All sheets, which you
receive, should be handed in, even if blank. Your 3-digit ID number is located on your
envelope. All problems carry the same weight.

________________________________________________________________________

1. A light spring hangs vertically from the ceiling. When a block of mass m is
attached to it the spring stretches downward a distance b, in equilibrium. If
the block is pulled down a distance d from this equilibrium position and
released from rest at time t=0, find the resulting motion of the block as a
function of time. This system is under the influence of earth’s gravity.

b
m
d

m
2. A string with unequal masses m1 and m2 hanging from its ends runs over a
massless and frictionless pulley of radius R. The axis of pulley is horizontal and
the masses may move in the vertical direction. The system is under the influence
of earth’s gravity.

R
g

m1

m2

What force F must be applied to the axis of the pulley to keep it stationary (no
motion up or down)?
3. (a) One of the following electrostatic fields is impossible. Which is it?
r
(1) E = k [xyxˆ + 2 yzyˆ + 3xzzˆ ]

r
[
(2) E = k y 2xˆ + (2 xy + z 2 )yˆ + 2 yzzˆ ]
where k is a constant with appropriate units; xˆ , yˆ , zˆ are unit vectors along the x,
y, and z axes, respectively. Assume there is no magnetic field.

(b) For the possible electric field, find the potential at an arbitrary point in space,
V(x, y, z). Assume the potential at the origin is zero, i.e. V(0, 0, 0) = 0.
4. A long solenoid carries a time-varying current that produces a magnetic field
B (t ) = B0 cos(ωt ) , where ω is the angular frequency. A single turn, circular loop
of radius of a/2 and a resistance R is placed inside the solenoid. A line through
the center of the solenoid passes through the center of the loop, but the axis of the
loop is inclined at an angle of θ with respect to the axis of the solenoid.

Solenoid
θ
a

a) Find the current induced in the circular loop.

b) Find the average power dissipated in the loop.


5. Consider a 1 dimensional infinite square well potential

⎧ 0 ; 0< x<a

V ( x) = ⎨
⎪⎩ ∞ ; otherwise

The wavefunctions for a single particle in this potential well are given by

2 ⎛ nπx ⎞
ψ n ( x) = sin ⎜ ⎟ , where n = 1, 2, 3 ...
a ⎝ a ⎠

What are the normalized wavefunctions that describe two particles confined to
this potential well in states defined by the quantum numbers n, m under the
following conditions?

(a) Each particle is distinguishable.


(b) The particles are indistinguishable fermions with their spins aligned along
the same axis.
(c) The particles are indistinguishable bosons.
6. A spin ½ particle with magnetic moment μ is located in a magnetic field oriented
in the +z direction.

a) A measurement at time t = 0 determines that the particle has its spin aligned
along the +z axis. What is the probability of observing the particle in this
same state at a later time t > 0 ?

b) A measurement at time t = 0 determines that the particle has its spin aligned
along the +x axis. What is the probability of observing the particle in this
same state at a later time t > 0 ?

Note: In the representation defined by the basis states with spin aligned in the +z and
–z directions, the operator that rotates a state by an angle θ about the y-axis is given
by:
⎛ cosθ / 2 − sin θ / 2 ⎞
ℜ y (θ ) = ⎜⎜ ⎟⎟
⎝ sin θ / 2 cosθ / 2 ⎠
7. An ideal gas engine that works according to the following cycle.

P
Isotherm at
P3 temperature T

P2

V2 V1 V

Find the efficiency of this engine assuming that the heat capacities of the gas may be
taken to be constant. Recall that the efficiency may be defined as

Net work done over full cycle


η=
Heat absorbed along isotherm

Express your answer in terms of the volumes V1 and V2 and the pressures P2 and P3
and the heat capacities at constant pressure and volume Cp and Cv.
7. An open glass tube of uniform bore (uniform inner diameter) is bent into the
shape of an “L”. One arm is immersed into a liquid of density ρ and the other
arm of length l remains in the air in a horizontal orientation. The tube is rotated
with constant angular speed ω about the axis of the vertical arm. Show that the
height y to which the liquid rises in the vertical arm is

P0 ⎛⎜ ⎡ ω 2l 2 M ⎤ ⎞
y= 1 − exp ⎢− ⎥⎟
gρ ⎜ ⎢⎣ 2 RT ⎥⎦ ⎟⎠

where P0 is the atmospheric pressure and M is the molecular weight of air and R
is the universal gas constant. Assume that the air pressure just inside open end of
the tube is atmospheric pressure, as shown.

l
P=P0

y
Physics Graduate School Qualifying Examination
Spring 2007 Part I
________________________________________________________________________

Instructions: Work all problems. This is a closed book examination. Calculators may
not be used. Start each problem on a new pack of correspondingly numbered paper
and use only one side of each sheet. Place your 3-digit identification number in the
upper right hand corner of each and every answer sheet. All sheets, which you
receive, should be handed in, even if blank. Your 3-digit ID number is located on your
envelope. All problems carry the same weight.
• Correct answers without adequate explanation/reasoning will not get full credit.
• Explain all variables you use in your derivations.
• Correct answer with incorrect reasoning will be counted wrong. Cross out
anything you don’t want us to read.
• Use correct vector notation when appropriate.
• Your work must be legible and clear.
________________________________________________________________________

1. A bullet of mass m1 strikes and embeds itself in the wooden block of mass m2 that
is suspended by massless strings from a ceiling, i.e. a ballistic pendulum. The
wooden block is initially motionless and hanging straight down as shown.
Immediately after the collision the block and the bullet both have a final velocity
of vf. The block and bullet continue to move until they have swung to an extreme
position where the wooden block and bullet are motionless (in the position
indicated by the dashed-line drawing). The final position of the wooden block is
a vertical distance h above the original position. The earth’s gravity acts in the
vertical direction.

m1 + m2

v1 m2 h
m1 vf

What is the initial speed of the bullet v1?

1
2. A loop of wire is bent into the shape of a parabola that is described by the
equation:

⎛ x2 ⎞
z = 1⎜ − b2 ⎟
2 ⎜ b2 ⎟
⎝ ⎠

where b is a constant. A bead of mass m is allowed to slide without friction on


the wire under the influence of gravity (gravity points in the –z direction). The
bead is released from rest at a height of zo above the bottom of the loop.

zo
x

(a) Calculate the velocity of the bead when it reaches the bottom of the loop
(x=0).

(b) Calculate the reaction force that the loop exerts on the bead, when the bead
is at the bottom of the loop.

2
3. Starting with the vector form of Coulomb’s law, find the electric field anywhere
on the z-axis due to a circular loop of thin wire that is uniformly charged with a
total charge of q. The loop has a radius of a and its center is at the origin of the
coordinate system.

a
y
.

3
4. Starting form the Biot-Savart Law, determine the magnetic field at an arbitrary point
on the z axis due to a circular current loop of radius a carrying constant current i in the
direction shown. The center of loop is at the origin of the coordinate system.

a
φ y
i

4
5. A quantum particle of mass m has the normalized wave function in the x-
coordinate,

⎧ 2 ⎛ πx ⎞
⎪⎪ sin ⎜ ⎟ ; 0≤ x≤L
ψ ( x) = ⎨ L ⎝L⎠
⎪ 0 ; elsewhere
⎪⎩
where L is a given constant length.

If px is the linear momentum of the particle give expressions for and calculate:

(a) <px>

(b) <(px)2>

(c) (Δpx)2

∫ sin ∫
A useful integral: n ( x)dx = − 1 sin n −1( x) cos( x) + n −1 sin n − 2 ( x)dx
n n

5
6. The operator for Lz , in spherical coordinates (r,θ,φ), is given by,


L z = − ih
∂φ

a) The physically correct eigenfunctions f(φ) of this operator are given


im φ
by f (φ ) = e , where m is an integer.

Explain why m must be an integer.

b) Suppose our wavefunction is given by,

ψ ( r , θ , φ ) = g ( r , θ )( 2 e i φ + 3e − 5 i φ ) .

If we measure Lz , what possible values could we find that have a nonzero


probability? What are the probabilities for each possible value?

6
7. (a) A one-dimensional system of ideal gas particles, each particle with a mass of
m, is at a temperature T. The gas is subjected to an external force whose potential
energy has the form

U ( x ) = Ax n ; 0 ≤ x ≤ ∞

with A>0 and n>0. Find the average potential energy per particle.

(b) What is the average potential energy per particle of an ideal gas in a uniform
gravitational field, i.e. U(x)= mgx?


Γ (1 / p )

p
[A useful Integral e − az dz ≡ ]
pa (1 / p )
0

7
8. A long cylindrical tank of length l and whose ends have a cross-sectional area
A has a mass when empty of M. The tank is filled with an ideal gas of mass Mg
at a pressure of Po and temperature of To. The total mass of the tank plus gas is
M+Mg. The tank is initially motionless but is resting on rails along which it
can move without friction. The rails are parallel to the axis of the cylinder.
The original position of the center of mass of the tank plus gas is in the middle
cylinder as shown.

y original
center of mass

l/2 x

Now the left end of the cylinder is held at the temperature To while heat has been
applied to the right end so that it is held at the temperature of To+Δt; Δt<<To.

(a) What is the new pressure in the tank? State any assumptions you make
about the temperature gradient in the tank.
(b) What is the new center of mass of the tank plus gas with respect to the
coordinate system attached to the tank (shown above)?
(c) How far has the tank moved relative to the ground?

8
Physics Graduate School Qualifying Examination
Spring 2007

Part II
________________________________________________________________________

Instructions: Work all problems. This is a closed book examination. Calculators may
not be used. Start each problem on a new pack of correspondingly numbered paper
and use only one side of each sheet. Place your 3-digit identification number in the
upper right hand corner of each and every answer sheet. All sheets, which you
receive, should be handed in, even if blank. Your 3-digit ID number is located on your
envelope. All problems carry the same weight.

________________________________________________________________________

1. An object of mass m is projected vertically upward from the surface of the


Earth. The Earth has a mass of ME and a radius of R. The initial speed of the
object is vo. Neglecting air resistance,

(a) Find the speed at any distance H above the Earth’s surface.

(b) Find the smallest initial velocity for which the object never falls back to
the surface of the Earth.

1
2. The wheel of a car, mass m, can be modeled as a uniform disk of radius a and
thickness b. When the car drives over a tremendous pothole, a small amount of
mass, Δm, located on the rim is displaced radially inward by an amount Δa as
shown:

Δm initial position
y Δm final position Δa

a
x

(a) If the inertia tensor of the undeformed wheel is

⎛ 3a 2 + b 2 0 0 ⎞⎟

m⎜ ⎟
I= 0 3a + b 2
2
12 ⎜ ⎟
⎜ 0 0 6a 2 ⎟
⎝ ⎠

calculate the inertia tensor after the wheel has been damaged, ignoring terms
that are of the order of (Δa)2 and higher.

(b) If the deformed wheel is constrained to rotate about the z-axis with angular
v v
velocity ω = ωk̂ , calculate the angle, α, between ω and the angular
momentum vector.

2
3. A charged particle of mass m and charge q is released at a short distance above
the surface of the Earth. The particle experiences a homogeneous magnetic field
r
B with orientation perpendicular to the force of gravity, e.g. +x-direction. The
r r
particle is also subject to a viscous force given by Fviscous = −kv , where
r
v =| v | is the speed of the particle.

r
B m,q

-mg

(a) Find the maximum speed v that can be reached by the particle.

(b) How far below its initial position (i.e., Δy) can the particle fall 1) when there
is a viscous force and 2) when there IS NOT a viscous force?

3
4. Please provide physical reasoning for your answers. r
E

(A, 2 pts) A hemispherical shell of radius R sits above the z=0 plane,
r
its center at the origin. A uniform field E points along the +z axis.
Find the magnitude of electric field flux passing through the shell.

R
(B, 2 pts) An uncharged metal spherical ball of radius R has
a spherical cavity of radius R/4 inside as shown. A point
charge q is placed into the center of the cavity. Find the net
charge on the outer surface of the ball. R/4

r
(C, 2 pts) A point object of mass m and net charge q>0 flying with velocity v enters the
r r r
area of uniform magnetic field B pointing out of page as shown by shaded area, v ⊥ B .
Assume that the shaded area occupies infinite space to the right of the division plane
shown by the dashed line. r r r
B=0 B
(i) Sketch the trajectory of the object as it passes
through the shaded area

(ii) How much work will be done by the magnetic r


force on the charge? B

(D, 2 pts) A student measured a magnetic field in some region of space which contains a
complicated electrical circuit and stated that he found the magnetic field to be:
r
B(x, y, z ) = ,0,0 , where a is a nonzero constant
a
x
Using Maxwell equations prove that student made a mistake.

(E, 2 pts) r r
A circular loop of radius r is made out of a wire with total B B
resistance R and mass m. At time t=0 it is released in the
r
gravitational field g of Earth and starts to fall with its r
r
plane being perpendicular to a uniform magnetic field, B .
r
g
Find current I flowing through the loop at time t>0.

4
5. Consider a 1 dimensional infinite square well potential

⎧ 0 ; 0< x<a

V ( x) = ⎨
⎪⎩ ∞ ; otherwise

A particle of mass m is confined in this potential and at time t=0 it has the
normalized wave function,
8 ⎡ ⎛ πx ⎞ ⎤ ⎛ πx ⎞
ψ ( x, t = 0) = ⎢1 + cos ⎜ ⎟ ⎥ sin ⎜ ⎟
5a ⎣ ⎝ a ⎠⎦ ⎝ a ⎠

(a) What are the possible energies of the particle described by this
wavefunction and the associated probability amplitudes for having these
energies?

(b) What is the time dependent wavefunction for this particle?

[ Useful Identity: cos α sin α = 1 sin( 2α ) ]


2

5
6. A particle of mass M moves in one dimension in the following potential:

⎧ 0 ; x<0

V ( x) = ⎨
⎪⎩ Vo ; x ≥ 0

V(x)

Vo

x
x=0

The particle has an energy E >Vo. The wavefunction is as follows:

⎧ Aeikx + Be−ikx ; x < 0



ψ ( x) = ⎨
⎪ Ceigx ; x ≥ 0

a) What are the values of k and g? (Express your answer in terms of E and Vo .)

b) Using the boundary conditions at x=0, find C/A as a function of E and Vo.
Your final answer for C/A should not contain B.

6
7. An ideal gas engine that works according to the following cycle.

P
P3
Isotherm at
temperature T

P2

V2 V1 V

Find the efficiency of this engine assuming that the heat capacities of the gas may be
taken to be constant. Recall that the efficiency may be defined as

Work output
η=
Heat absorbed

Express your answer in terms of the volumes V1 and V2 and the pressures P2 and P3
and the heat capacities at constant pressure and volume Cp and Cv.

7
8. Consider a centrifuge of radius a. The centrifuge contains an ideal gas of particles
of mass m, at a temperature T. When centrifuge is at rest, the gas has a density of
no.

When the centrifuge is spinning with a constant angular velocity ω, calculate


the density profile as a function of the radial variable r, i.e. n(r).

8
Physics Graduate School Qualifying Examination
Fall 2007 Part I
________________________________________________________________________

Instructions: Work all problems. This is a closed book examination. Calculators may
not be used. Start each problem on a new pack of correspondingly numbered paper
and use only one side of each sheet. Place your 3-digit identification number in the
upper right hand corner of each and every answer sheet. All sheets, which you
receive, should be handed in, even if blank. Your 3-digit ID number is located on your
envelope. All problems carry the same weight.
• Correct answers without adequate explanation/reasoning will not get full credit.
• Explain all variables you use in your derivations.
• Correct answer with incorrect reasoning will be counted wrong. Cross out
anything you don’t want us to read.
• Use correct vector notation when appropriate.
• Your work must be legible and clear.
________________________________________________________________________

1) Consider two identical with masses M on springs of force constant k that are
attached to fixed walls. The two masses are connected by a spring of force constant
k′ .

Calculate the fundamental frequencies of the coupled system and interpret.

1
2) A thin triangular lamina of mass M with dimensions as shown below, is hit by a small
G
ball of putty of mass m << M which moves with velocity v = vî
y

G
v = vî

x
a

Before the collision, the position of the center-of-mass of the lamina is located at

xcm = a/3
ycm = a/3

and the moment of inertia about its center of mass (rotation axis parallel to z-axis) is

Icm = Ma2/9

(a) If the putty hits the corner of the lamina at the point y = a and sticks there, calculate
the velocity of the center-of-mass of the lamina after the collisions.

(b) Calculate the angular velocity of the lamina about its center of mass after the
collision.

Assume that the mass of the putty is small enough that its contribution to moment of
inertia of the lamina + putty system is negligible.

2
3) A proton moves in a circle of radius r in a constant magnetic field that is
perpendicular to the plane of the circle. The magnitude of the magnetic field is B.

a) Calculate the angular frequency at which the proton moves around the circular
path.

b) What is the kinetic energy of the proton?

G G G
Hints: Fc = mv 2 / r ; FB = q(v × B)

3
4. Suppose we have a time-independent current density given by
G G
J (r ) = azzˆ

where a is a constant. Let Q be the total charge contained in the cube bounded by 0<x<L,
0<y<L, 0<z<L.

L
y
L

dQ
Compute .
dt

4
5) At time t=0, a particle is in a state which is a linear superposition of two energy
eigenstates, i.e. the normalized wavefunction of the particle is given by,

ψ ( x) =
1
(ψ 1 ( x) + ψ 2 ( x) )
2
where ψ i ( x) are the normalized wavefunctions, i= 1 or 2, of the energy eigenstates.
These energy eigenstates have energy eigenvalues of Ei , i= 1 or 2.

Suppose a measurable physical quantity of the particle is described by the time


independent quantum operator  . Assume that the expectation value of  is given by
A(t).

a) What is the wavefunction of the particle at time t=T?

b) In this superposition state, the expectation of  , A(t), is expected to be a


periodic function of t. What is the period?

c) Derive a condition so that A(t) is time independent.

5
6) The spin functions for a free electron in a basis where the spin operator ŝ z is diagonal
can be written as

⎛1⎞ ⎛ 0⎞
⎜⎜ ⎟⎟ and ⎜⎜ ⎟⎟
⎝ 0⎠ ⎝1⎠

with the eigenvalues of ŝ z (taking = =1) of these states being +1/2 and -1/2 ,
respectively.

Using this basis find the normalized eigenfunction of ŝ y with eigenvalue -1/2. Also,
find the probability that sz = +1 in that state.

1 ⎛0 1⎞ i ⎛ 0 − 1⎞ 1 ⎛1 0 ⎞
sˆ x = ⎜⎜ ⎟⎟ ; sˆ y = ⎜⎜ ⎟⎟ ; sˆ z = ⎜⎜ ⎟
2 ⎝1 0⎠ 2 ⎝1 0 ⎠ 2 ⎝ 0 − 1⎟⎠

6
7. An ideal gas is in equilibrium at a given initial temperature T0, initial pressure P0,
and volume V0. The volume of the gas is increased by the infinitesimal amount
δV=V-V0 such that its pressure changes by δP = b×δV. Note that b is a constant
which can be either positive, negative, or zero (in the diagram below it is negative).

For what values of b is δT positive?

7
8. Classically, the fusion of 2 protons (ionized H) in a hot gas is hindered by the
electrostatic repulsion between the 2 charged particles.

(a) Calculate the gas temperature required for 2 protons to overcome the
electrostatic repulsion between them and fuse at a separation distance of r .

From a quantum mechanical point of view the fusion of 2 protons can occur when
the distance between them is comparable to their DeBroglie wavelength (i.e. they
can tunnel through the electrostatic barrier at that distance).

(b) Find an expression for the DeBroglie wavelength (λ) for which the kinetic
energy of a proton is equal to the electrostatic potential energy between two
protons when they are separated by this same λ. Then use this separation
distance λ to find the temperature of a gas that allows fusion to occur.

8
Physics Graduate School Qualifying Examination
Fall 2007 Part II
________________________________________________________________________

Instructions: Work all problems. This is a closed book examination. Calculators may
not be used. Start each problem on a new pack of correspondingly numbered paper
and use only one side of each sheet. Place your 3-digit identification number in the
upper right hand corner of each and every answer sheet. All sheets, which you
receive, should be handed in, even if blank. Your 3-digit ID number is located on your
envelope. All problems carry the same weight.
• Correct answers without adequate explanation/reasoning will not get full credit.
• Explain all variables you use in your derivations.
• Correct answer with incorrect reasoning will be counted wrong. Cross out
anything you don’t want us to read.
• Use correct vector notation when appropriate.
• Your work must be legible and clear.
________________________________________________________________________

1. Consider a mass M suspended between two identical springs as shown below (left).

a) Calculate the force on the mass exactly for displacements along the mid-
plane (above right), if the springs are initially free from tension. (neglect
gravity)

b) What is the form of the force for small displacements? In this limit, is the
system linear?

1
2. An engine pulls a train in which a train car carrying toxic goo has sprung a leak as
shown. If the engine applies a constant force F to the train which has an initial mass,
M, calculate the velocity of the train as a function of time, assuming that the toxic goo
leaks out at a constant rate.

2
G G G
3. In a perfect conductor, the conductivity is infinite, so E = 0 ( j = σE ) , and any net
charge resides on the surface (just as it does for an imperfect conductor in
electrostatics).

G
⎛ ∂B ⎞
(a) Show that the magnetic field is constant ⎜⎜ = 0 ⎟⎟ inside a perfect
⎝ ∂t ⎠
conductor.

(b) Show that the magnetic flux through a perfectly conducting loop is
constant.

A superconductor is a perfect conductor with the additional property that the


(constant) magnetic field inside is zero. (This flux exclusion is known as the
Meissner effect.)

(c) Show that the current in a superconductor is confined to the


surface.

3
4. The length of a cylindrical solenoid (L) is much larger than its radius r. The coil is
made of a wire of square cross-section with a size a<r, the coil is tightly wound. When
current is passed through the coil the solenoid feels a radially outward pressure due to a
magnetic force acting radially outward on the wire. Calculate this pressure.

Single Coil (end view) G


Fradial
G
Fradial
G
Fradial

G
Fradial a

G
Fradial
G
Fradial

4
5. Two quantum mechanical particles have the same mass, m, and are in a one
dimensional space. An attractive force between them grows linearly as the separation
distance between them, therefore, the total system is described by the following
Hamiltonian

p12 + p22 1
H = + k ( x1 − x2 ) 2
2m 2

Where p1, p2, x1, and x2 are the momenta and positions of the two particles, respectively.
k is a constant.

(a) What is the ground state energy of the system if the two particles are
identical bosons?

(b) What is the ground state energy of the system if the two particles are
identical spin 1/2 fermions in a triplet state?

5
6. A particle of mass m is incident from the right on the step potential given below:

⎧ 0 ; x<0

V ( x) = ⎨
⎪⎩ Vo ; x ≥ 0

V(x)

Vo

x
x=0

The particle has an energy of E>V0 . As the particle comes in from the right
what is:

a) The reflection coefficient from this step potential? That is the ratio of
the reflected current to the incident current.

b) The transmission coefficient from this step potential? That is the


ratio of the transmitted current to the incident current.

6
7. Stars are radiating gas spheres in a state of hydrostatic equilibrium (i.e. the pressure
forces are in balance with the gravitational forces at every point within the star). As such,
stars are governed by the virial theorem which states that the sum of twice the total
thermal energy and the total gravitational potential energy vanishes. That is

2U + Egrav = 0.

(a) Find an expression for the total gravitational potential energy of the Sun. Assume the
density of the Sun is constant. Write your answer in terms G, Msun , and Rsun (the
Universal Gravitational Constant, the mass of the Sun, and the radius of the Sun,
respectively).

(b) Then using that expression and the virial theorem find the average temperature of the
Sun. In addition to assuming a constant density for the Sun, also assume that it is an
ideal gas composed entirely of hydrogen (H) which has a mass mH.

7
8. A monatomic ideal gas works according to the cycle shown in the figure.

a. Calculate the total work done over the entire cycle.


b. Calculate in each of the segments the:
i. Net heat absorbed in 1 to 2
ii. Net heat absorbed in 2 to 3
iii. Net heat absorbed in 3 to 1

8
Physics Graduate School Qualifying Examination
Fall 2008 Part I
________________________________________________________________________

Instructions: Work all problems. This is a closed book examination. Calculators may
not be used. Start each problem on a new pack of correspondingly numbered paper
and use only one side of each sheet. Place your 3-digit identification number in the
upper right hand corner of each and every answer sheet. All sheets, which you
receive, should be handed in, even if blank. Your 3-digit ID number is located on your
envelope. All problems carry the same weight.
• Correct answers without adequate explanation/reasoning will not get full credit.
• Explain all variables you use in your derivations.
• Correct answer with incorrect reasoning will be counted wrong. Cross out
anything you don’t want us to read.
• Use correct vector notation when appropriate.
• Your work must be legible and clear.
________________________________________________________________________

1. A uniform stick of mass M and length L is suspended from a ceiling by two


massless strings at its ends.

Uniform
Gravitational
Field
G
g
M
L

One of the strings is cut. Immediately after being cut,

a) What is the torque about the end of the stick attached to the uncut string?

b) What is the angular acceleration about the end of the stick attached to the
uncut string?

c) What is the vertical acceleration of the center of mass?

d) What is the vertical force on the uncut string?

(The moment of inertia of a stick about one end is 1/3 ML2)

1
2. Consider two masses joined by a string as shown:

Uniform Gravitational
m Field
G
g
R
θ
M

The heavy mass M is also attached to the rim of a pulley wheel of radius R. Assume the
system is in a uniform gravitational field (i.e. on the surface of the earth). Ignore the
mass of the string and the moment of inertia of the wheels.

a. What is the kinetic energy of the system in terms of the vertical position
y of the small mass m, the rotation angle θ, and the masses m and M?

b. What is the potential energy of the system in terms of y and θ ?

c. What equation relates y and θ ?

d. Write the Lagrangian for the system and derive the equation of motion.

e. What is the equilibrium position, θ0, at which θ = θ = 0 ?

f. Express the rotation angle as θ = θ 0 + δ where δ << 1 and expand the


Lagrangian to second order in δ.

g. By comparing this Lagrangian from (f) with the Lagrangian for a


harmonic oscillator,
1 1
L = mx 2 − kx 2
2 2

which has an oscillation frequency of ω = k / m , show that the frequency


of small oscillations for the mass M is

g M −m
ω2 = .
R M +m

2
3. Two thin-walled, cylindrical, metal tubes have radii a and b (a < b). The tube with
the smaller radius is inserted coaxially into the other tube. Both tubes stand vertically in
a tank of dielectric oil. The oil has a dielectric constant of ε and a density of ρ. A
potential voltage of V is applied between the tubes.

V
Uniform Gravitational
Field
G
g

b
a
h

To what height, h, does the oil rise in the space between the tubes?

3
4. A circular ring of radius, a, is uniformly charged with total charge q.
z

a) Find the electric potential on the symmetry axis z that is perpendicular to the
plane of the ring and passes through its center.

b) For a cylindrically symmetric charge distribution, the potential at large r ( r


large compared to any dimension of the charge distribution) is
approximately (to third order in 1/r),

1 1 1
V (r ,θ ) = A0 P0 (cos θ ) + A1 P1 (cos θ ) + A2 P2 (cos θ )
r r2 r3

= A0
1
r
+ A1
r
1
2
cos(θ ) + A2
1 ⎡1
r ⎣2
3 ⎢ ( )⎤
3 cos 2 (θ ) − 1 ⎥

where r 2 = x 2 + y 2 + z 2 and z =rcos(θ). The An are constant coefficients.


Use your result from part (a) to determine these coefficients.

4
G
5. Consider a spin ½ atom in a magnetic field of B = Bzˆ . The Hamiltonian is
G G G
H = − μB ⋅ S , where μ is the magnetic moment and S is the spin operator defined:

= ⎛0 1⎞ = ⎛0 − i⎞ = ⎛1 0 ⎞
Sx = ⎜⎜ ⎟⎟ ; S y = ⎜⎜ ⎟⎟ ; S z = ⎜⎜ ⎟
2 ⎝1 0⎠ 2⎝i 0 ⎠ 2 ⎝ 0 − 1⎟⎠

a) Find the eigenvectors and eigenvalues of H.

b) In the basis of the eigenvectors of H, find an eigenstate of Sx with an


eigenvalue of + 1 = .
2

c) Describe qualitatively (and briefly) the time evolution of an atom that is


initially in the state you found in (b).

5
6. Consider a particle of mass m moving, in one dimension, in a square well potential of
width 2a and depth Vo as in the figure below.
V(x)

x= -a x=a x

Vo

Namely, the Hamiltonian is

p2 ⎧ − Vo ; x < a
H= + V ( x) with ⎪ .
2m V ( x) = ⎨
⎪ 0 ; x >a

Consider now the following wave-function

⎧ α 2 cos kx ; x < a
⎪⎪ π π 1
where k = K = , α= .
ψ ( x) = ⎨ K (a − x ) 4a 2a 4+π
⎪ αe ; x >a
⎪⎩

Assuming that ψ (x) is a normalized eigenfunction of the Hamiltonian:

a) Compute, in terms of m and a, the energy eigenvalue of ψ (x) .

b) Compute, in terms of m and a, the depth of the potential, that is Vo .

c) Compute the probability of finding the particle outside the square


well, i.e. in the region |x| > a.

6
7. Consider a classical system with N (N>>1) particles. There are two energy levels
for each particle. The energy is equal to zero for a particle in the ground state and E1
for a particle in the excited state. Moreover, the total energy of the system, E, can be
measured very accurately.

Can this system be used to design a thermometer? If no, state the reason. If yes,
provide the formula to determine the temperature as a function of E and N.

Recall that ln(N!) ≅ N ln(N) – N .

7
8. On February 23rd, 1987 a supernova was observed in the Large Magellanic
Cloud a distance of D=170,000 light years from Earth. In a tank of 1000 tons
of water beneath a mountain in Japan 10 neutrinos from the supernova were
observed to arrive over a time interval of Δt=1 second. The average energy of
the neutrinos was 10 MeV and the range of energies was from E1=5MeV to
E2=20MeV.

Estimate an upper limit on the neutrino mass. Assume the neutrinos are
emitted at the same time and place. Your answer should be in terms of
the observed arrival time interval Δt, the distance traveled D, and the
limits on the range of energy E1 and E2.

8
Physics Graduate School Qualifying Examination
Spring 2008 Part I
________________________________________________________________________

Instructions: Work all problems. This is a closed book examination. Calculators may
not be used. Start each problem on a new pack of correspondingly numbered paper
and use only one side of each sheet. Place your 3-digit identification number in the
upper right hand corner of each and every answer sheet. All sheets, which you
receive, should be handed in, even if blank. Your 3-digit ID number is located on your
envelope. All problems carry the same weight.
• Correct answers without adequate explanation/reasoning will not get full credit.
• Explain all variables you use in your derivations.
• Correct answer with incorrect reasoning will be counted wrong. Cross out
anything you don’t want us to read.
• Use correct vector notation when appropriate.
• Your work must be legible and clear.
________________________________________________________________________

1. Three identical point masses are located at (a,0,0 ) , (0,2a, a ) and (0, a,2a ) where
the ordered triple represents (x,y,z).

Calculate the moment of inertia tensor about the origin, the principle moments
and a set of principle axes, i.e. the eigenvalues and eigenvectors of the moment
of inertia tensor.

1
2. A pendulum consists of a sphere of radius r and mass M hanging from a string of
length R. This particular pendulum hangs over an inclined plane so that it rolls without
slipping on the inclined surface as shown below. Earth’s gravity points vertically down
the page. You may ignore the torsional effect of the twisting string. Also the moment of
inertia of a sphere is I=2/5Mr2.
Side View
gravity

θ φ
φ
r
sphere rotates through
angle ψ as it rolls
without slipping.

a. What is the kinetic energy of the system in terms of the angle θ and the
angle ψ through which the sphere has rotated as it rolled?

b. What is the potential energy of the system in terms of θ and ψ?

c. What equations relate θ and ψ?

d. Write the Lagrangian for the system to second order in θ , i.e. assume θ
is a small angle.

e. By comparing the Lagrangian in (d) with the Lagrangian for a harmonic


oscillator,

1 2 1 2
L= mx − kx
2 2

which has an oscillation frequency of ω = k / m , what is the frequency of


small oscillations of the rolling pendulum?

2
3. Two identical spheres of radius r are separated by a distance d >> r. A charge Q is to
be placed on the surface of the spheres.

a) What is the potential energy of the system if we put Q/2 on each sphere?

b) What is the potential energy if we put all the charge on one sphere?

c) What are the electric potentials on each sphere in cases a) and b)?

d) If we were to connect the spheres in b) with a fine wire so that the charge
could flow from one to the other, what would be the final charge
configuration?

e) Is electrostatic energy conserved in the process described in part d)? Is the


total energy conserved in the process?

3
4. Inside a superconductor, the electrodynamics are described by London’s equation,
G G G G
A = −λJ (assumes the gauge choice of ∇ D A = 0 )
G G
where A is the vector potential and J is the electric current density.

Consider an infinite superconducting slab of thickness 2d, ( − d ≤ z ≤ d ), outside of


which there is a given constant magnetic field parallel to the surface:

B x = B z = 0 , B y = Bo
G
with the electric field E = 0 everywhere.
G G
Calculate the magnetic field B and current density J inside the slab. What is the
magnetic field deep inside the slab when d is very large?

4
5. Consider the “double square well” potential shown below. Assume the depth, Vo, and
the width, a, are large enough that several bound states exist.

x
a a -Vo
b

Sketch the ground state wavefunction, ψ1, and the first excited state, ψ2, for the 3
cases:

i) b=0 ;
ii) b~a ;
iii) b>>a.

5
6. Consider the asymmetric one-dimensional infinite potential well:

⎧ 0 ; 0≤ x≤a

V ( x) = ⎨
∞ ; otherwise

a) Find the energy eigenfunctions for a single particle of mass m in the


well.

b) If there are two identical, non-interacting bosons of mass m in the


well, find the energy eigenfunctions and eigenvalues for the ground
state and the first excited state.

6
7. A monatomic ideal gas works in a heat engine according to the cycle shown.
Process 1->2 is an isotherm, 2->3 a constant volume process and 3->1 is an adiabatic
expansion.

a) Find the efficiency of this cycle. The efficiency is defined as the ratio of the
work done by the gas over the entire cycle to the absorbed heat. Express the
result in terms TL=T1=T2 and TH=T3.

b) Compare the result to that of a reversible Carnot cycle running between


reservoirs of TL and TH.
(Reminder: A Carnot cycle consists of two adiabats and two isotherms)

adiabatic
expansion
2

isothermal 1
V

7
8. Consider a monatomic gas with no interactions between the atoms and a kinetic
energy per atom of λp, where λ is a constant and p is the momentum.

(a) Show whether this gas does or does not obey the ideal gas law
( PV = Nk B T , P is the pressure, V is the volume, N is the number of atoms,
kB is Boltzmann’s constant, and T is the temperature).

(b) Find U(N,V,T).

(c) Find CV(N,T).

8
Physics Graduate School Qualifying Examination
Fall 2008 Part II
________________________________________________________________________

Instructions: Work all problems. This is a closed book examination. Calculators may
not be used. Start each problem on a new pack of correspondingly numbered paper
and use only one side of each sheet. Place your 3-digit identification number in the
upper right hand corner of each and every answer sheet. All sheets, which you
receive, should be handed in, even if blank. Your 3-digit ID number is located on your
envelope. All problems carry the same weight.
• Correct answers without adequate explanation/reasoning will not get full credit.
• Explain all variables you use in your derivations.
• Correct answer with incorrect reasoning will be counted wrong. Cross out
anything you don’t want us to read.
• Use correct vector notation when appropriate.
• Your work must be legible and clear.
________________________________________________________________________

1. A chain with a mass per unit length of σ and total length L hangs with one end just
touching a scale. The chain is released from rest. As the chain falls on to the scale the
force that is registered by the scale will change.

Uniform Gravitational
L Field
G
g

What is the force registered by the scale as a function of y, the height of the chain
that remains above the scale, i.e. F(y)? (The scale is on the surface of the earth.)

1
2. A massless ring of radius R hangs from the ceiling by a thread. Two beads of mass m
slide on the ring without friction. The beads are released simultaneously from the top and
slide down on opposite sides of the ring (as shown below). At what angle θ will the ring
begin to rise?

ωo

Uniform Gravitational
m m Field
G
θ g

2
3. A point charge of +Q is located a distance d from the center of a grounded, conducting
spherical shell of radius R. The charge lies on the z-axis.

θ +Q
z
d

Find the electric potential at the arbitrary point P in spherical polar coordinates,
V (r ,θ , φ ) for r ≥ R .

3
4. A circular disk rotates about its axis with angular velocity ω. The disk is made of
metal with conductivity σ, and its thickness is t. The rotating disk is placed between the
G
pole faces of a magnet which produces a uniform magnetic field, B , over a small square
G
area of size a2. The center of the square region is at the distance of r from the axis; B is
perpendicular to the disk.

a
a r ω
t

square area
with uniform
magnetic field

Calculate the approximate torque on the disk.

4
5. Consider a rigid body made up of two (non-identical) particles of mass m attached
to the ends of a massless rigid rod of length a. The system rotates in the x-y plane
about a vertical z-axis through its stationary center of mass. Use φ to denote the
rotation angle.
z
a

m m

a) Find the energy eigenvalues and the corresponding eigenfunctions of the


body.
b) If at time zero (t=0) ψ (φ , t = 0) = (1 − cos(2φ ) ) , find ψ (φ , t ) .
A
2

5
6. Consider the infinite, one-dimensional potential well:


⎪ ∞ ; x<0
∞ ∞ ⎪

V ( x) = ⎨ 0 ; 0≤ x≤a
V(x) ⎪
⎪ ∞ ; x>a

x
x=0
x=a

a) Find the energy eigenvalues and energy eigenfunctions of a single


particle of mass m in the well, i.e 0 ≤ x ≤ a .

b) If there are two identical, non-interacting spin ½ fermions of mass m


in the well, find the ground state energy and the ground state
wavefunction. Use χ+(1) and χ-(1) as the spin up and spin down
eigenfunctions for particle one, respectively. Similarly, use χ+(2) and
χ-(2) be the spin up and spin down eigenfunctions for particle two,
respectively.

6
7. An engine operates in a cycle as follows: it extracts heat Q1 from a large energy
reservoir at temperature T1. It does work W, and puts heat Q2 into a large reservoir at
temperature T2 (with T2 < T1). At the end of the cycle, the engine has returned to its initial
state. Energy conservation then tells us that Q1 = W + Q2.

What is the maximum amount of work W that can be produced? Express your
answer in terms of Q1, T1 and T2. (Note that we are taking W, Q1, and Q2 to all be positive
and the temperatures are measured in Kelvins.)

T1

Q1

W
Engine

Q2

T2

7
8. Consider an interstellar gas cloud that has a mass M and that is at a temperature T.
It is composed of hydrogen atoms. The mass of a hydrogen atom is mH.

Estimate the density at which this gas cloud will condense into a star (assume a
spherical cloud of uniform density with a radius of R).

8
Physics Graduate School Qualifying Examination
Spring 2008 Part II
________________________________________________________________________

Instructions: Work all problems. This is a closed book examination. Calculators may
not be used. Start each problem on a new pack of correspondingly numbered paper
and use only one side of each sheet. Place your 3-digit identification number in the
upper right hand corner of each and every answer sheet. All sheets, which you
receive, should be handed in, even if blank. Your 3-digit ID number is located on your
envelope. All problems carry the same weight.
• Correct answers without adequate explanation/reasoning will not get full credit.
• Explain all variables you use in your derivations.
• Correct answer with incorrect reasoning will be counted wrong. Cross out
anything you don’t want us to read.
• Use correct vector notation when appropriate.
• Your work must be legible and clear.
________________________________________________________________________

1. A particle of mass m moves in a central-force field described by the potential


function U (r ) .

a. Express the Lagrangian for this system in plane polar coordinates


(r,θ).

b. From Lagrange’s equation for the angular coordinate, find an


expression for θ .

c. Using Lagrange’s equation for the radial coordinate, derive an


expression relating the central-force law F (r ) and the orbit r (θ ) .
(Hint: Make the change of variable u ≡ 1 r , and calculate du dθ and
d 2 u dθ 2 )

d. If the force law is given by F (r ) = − k r n , and the particle’s orbit is


circular and passes through the force center, find n . (Hint: the
formula for this orbit in plane polar coordinates is r (θ ) = A cos(θ ) ,
where A is the diameter of the orbit.)

1
2. A wheel of radius r and moment of inertia Io is spinning with an initial angular velocity
ωo as shown below:

ωo

spray gun

Top View
r ψ
ωo
spray gun

The wheel is painted by a spray gun which deposits additional mass on the rim of the
wheel at a constant rate. The paint in the spray has a velocity of v ( v >> ω o r ) and hits
the rim at an angle ψ as shown in the top view.

What is the angle ψ that will cause the angular velocity to remain ωo for all time?

2
3. A capacitor made of two parallel uniformly charged circular conducting metal disks
carries a charge of +Q and –Q on the inner surfaces of the plates, and very small amounts
of charge +q and -q on the outer surfaces of the plates. Each plate has a radius R and
thickness t, and the gap distance between the plates is s.

How much charge q is on the outside surface of the positive disk, in terms of Q? Do
NOT use Gauss’s law to solve this problem!

Hint: The electric field of a uniformly charged disk of radius R a distance z from the
center of the disk along a line perpendicular to the disk is given by:

(Q / A) z
E= [1 − 2 ], A = π R 2
2ε 0 (R + z )
2

t +Q

-q

-Q +q

3
4. The standard Maxwell’s equations posit no magnetic charge. When there is a magnetic
charge, the Maxwell equations become symmetric with respect to the electric and
magnetic fields. In the presence of a magnetic charge, the Gauss’s law for magnetism
becomes

G G ρ
∇DB = m
μo

where ρm is the density of the magnetic charge.

Let’s consider a point particle with electric charge Qe and velocity ve and
another point particle with magnetic charge Qm. What is the force on the
electrically charged particle when the magnetically charged particle is not
moving? For simplicity, assume the magnetically charged particle is at the
origin.

4
5. The potential for a one-dimensional harmonic oscillator ( V ( x ) = 1 kx 2 ) and the
2
first few energy eigenfunctions are shown in the diagram below. The energies of the
eigenstates are E n = ( n + 1 ) = ω .
2

Find the separation in energy for adjacent states of the half harmonic oscillator
(shown below) and explain your reasoning.


V(x)
⎧ 1 kx 2 ; x ≥ 0
⎪⎪ 2
V ( x) = ⎨
⎪ ∞ ; x<0
⎪⎩
1 kx 2
2

x=0 x

5
6. A quantum particle of mass m with an energy E<V0 is in the one-dimensional
potential:


⎪ ∞ ; x<0
∞ ⎪

V ( x) = ⎨ 0 ; 0 ≤ x ≤ a
V(x) ⎪
⎪ Vo ; x > a

Vo

x
x=0
x=a

a) Write Schrodinger’s Equation for the energy eigenfunctions in the


separate regions 0 ≤ x ≤ a and x>a.

b) Find the form of the energy eigenfunctions in these separate regions.

c) What conditions must be imposed on the energy eigenfunctions at


x=a? Note: you do not need to impose them or find the final overall
energy eigenfunctions.

6
7. An immersion heater of power P is used to heat water in a bowl. After a time t1, the
temperature of the water T increases by ΔT1. The heater is switched off and T drops by
ΔT2 in a time t2=t1/2. The temperature of the room To is much smaller than the
temperature of the water at all times, so that the rate of energy loss of the water to the
room can be assumed to be constant through out this process.

What is the mass m of the water in the bowl? (C is the specific heat of water)

7
8. A monatomic ideal gas of particles each of mass m is at a temperature T. The gas
is subject to an external force whose potential energy is


⎪ Ax n , 0 ≤ x ≤ ∞ .
U ( x) = ⎨

⎩ ∞, x < 0

A>0, and the integer n>0.

a) Find the average potential energy per particle.

b) What is the average potential energy per particle if the gas is in a uniform
gravitational field?


(n )
Hint: z1 / (n −1)e − z dz ≡ Γ 1

0

Potrebbero piacerti anche